Question game

Hii everyone. This note is a type of quiz game where you can solve questions and post them too. The advantage of this note is you can practice both solving and posting good questions. Your solutions and questions will be appreciated too by community members. So, enjoy the beauty of solving the questions. Go ahead :

Rules And Regulations :

  1. In this game the questions will be posted and by the community members itself.

  2. The one who solves the latest question first will be able to post a new question. You should provide a clear solution to the question you have solved. Only the one who first solves the latest question correctly will have a chance to post the new question.

  3. If any member has a different solution to a question which has already been solved he can also post the solution but he cannot post a new question.

  4. There is also a leader board (below) in which the top 10 highest solvers will be shown. So try to solve as many questions as possible.

  5. The one who posted the question should not be the first one to solve that question. However he can post his solution if he wants only after anybody solves his question.


Summary

Total Questions Solved : 6

Latest Question Posted : Problem 6

Leader Board (Top 10 highest Solvers) :

NAME - PROBLEMS SOLVED

11.Vilakshan Gupta - 2

22. Ram Mohith - 2

33. Vaibhav Priyadarshi and R Mathe and Michael Mendrin and Tom Clancy - 1

44. Matin Naseri - 1


If any improvements are required in this game kindly inform them to me.

Thank You and All the Best

#Algebra

Note by Ram Mohith
3 years ago

No vote yet
1 vote

  Easy Math Editor

This discussion board is a place to discuss our Daily Challenges and the math and science related to those challenges. Explanations are more than just a solution — they should explain the steps and thinking strategies that you used to obtain the solution. Comments should further the discussion of math and science.

When posting on Brilliant:

  • Use the emojis to react to an explanation, whether you're congratulating a job well done , or just really confused .
  • Ask specific questions about the challenge or the steps in somebody's explanation. Well-posed questions can add a lot to the discussion, but posting "I don't understand!" doesn't help anyone.
  • Try to contribute something new to the discussion, whether it is an extension, generalization or other idea related to the challenge.
  • Stay on topic — we're all here to learn more about math and science, not to hear about your favorite get-rich-quick scheme or current world events.

MarkdownAppears as
*italics* or _italics_ italics
**bold** or __bold__ bold

- bulleted
- list

  • bulleted
  • list

1. numbered
2. list

  1. numbered
  2. list
Note: you must add a full line of space before and after lists for them to show up correctly
paragraph 1

paragraph 2

paragraph 1

paragraph 2

[example link](https://brilliant.org)example link
> This is a quote
This is a quote
    # I indented these lines
    # 4 spaces, and now they show
    # up as a code block.

    print "hello world"
# I indented these lines
# 4 spaces, and now they show
# up as a code block.

print "hello world"
MathAppears as
Remember to wrap math in \( ... \) or \[ ... \] to ensure proper formatting.
2 \times 3 2×3 2 \times 3
2^{34} 234 2^{34}
a_{i-1} ai1 a_{i-1}
\frac{2}{3} 23 \frac{2}{3}
\sqrt{2} 2 \sqrt{2}
\sum_{i=1}^3 i=13 \sum_{i=1}^3
\sin \theta sinθ \sin \theta
\boxed{123} 123 \boxed{123}

Comments

This is a nice discussion. Really enjoyed

Syed Hamza Khalid - 2 years, 8 months ago

Problem 1 (solved by Vaibhav Priyadarshi)

Find the value of log210log8125\log_2 10 - \log_8 125

Ram Mohith - 3 years ago

Log in to reply

log210log8125log_{2}10-log_{8}125

=log210log23125log_{2}10-log_{2^3}125

=log210(log2125)/3log_{2}10-(log_{2}125)/3

=(3log210log2125)/3(3log_{2}10-log_{2}125)/3

=(log2103log2125)/3(log_{2}10^3-log_{2}125)/3

=(log2(1000/125))/3(log_{2}(1000/125))/3

=(log28)/3(log_{2}8)/3

=3/3 =1.

Vaibhav Priyadarshi - 3 years ago

Log in to reply

Now can I post a new question?

Vaibhav Priyadarshi - 3 years ago

Log in to reply

@Vaibhav Priyadarshi Yes you can post a new question

Ram Mohith - 2 years, 12 months ago

Please use \log in your solution.

Ram Mohith - 2 years, 12 months ago

Another solution for Problem 1 :

log8125=log23125=13log2125=log21253=log25\log_8 125 = \log_{2^3} 125 = \dfrac{1}{3}\log_2 125 = \log_2 \sqrt[3]{125} = \log_2 5

log210log8125=log210log25\log_2 10 - \log_8 125 = \log_2 10 - \log_2 5

    log2105=log22=1\implies \log_2 \dfrac{10}{5} = \log_2 2 = 1

Ram Mohith - 2 years, 12 months ago

Problem 2 :\textbf{Problem 2 :}

Find the value of (5+213)1/3+(5213)1/3(5+2\sqrt{13})^{1/3}+(5-2\sqrt{13})^{1/3}

Vaibhav Priyadarshi - 2 years, 12 months ago

Log in to reply

14(1+313+i(393))\frac{1}{4}(1+3\sqrt{13}+i(\sqrt{39}-\sqrt{3}))?

Michael Mendrin - 2 years, 12 months ago

Log in to reply

Sir, if this is correct you can post a new question but whether it is correct or not Vibhav should tell it.

Ram Mohith - 2 years, 12 months ago

Log in to reply

@Ram Mohith I don't think the answer is "1"

Michael Mendrin - 2 years, 12 months ago

Log in to reply

@Michael Mendrin Just manipulate the expression using algebra and you will see that...wow... it turns out to be 1!

Some things look complicated but just do the math and you will get the answer.

Vilakshan Gupta - 2 years, 12 months ago

@Michael Mendrin I will give you a hint , note that a3+b3=10a^3+b^3=10. Rest is upto you

Vilakshan Gupta - 2 years, 12 months ago

Log in to reply

@Vilakshan Gupta Vilakshan Gupta why can't you post your solution if you have solved

Ram Mohith - 2 years, 12 months ago

@Vilakshan Gupta (a3+b3)=(a+b)(a2ab+b2)=10(a^3+b^3)=(a+b)(a^2-ab+b^2)=10, but I don't see where the "1" comes up. Both factors are complex conjugates.

Michael Mendrin - 2 years, 12 months ago

Log in to reply

@Michael Mendrin Actually what I meant was Vaibhav's solution only, because if you cube the expression, the radicals get cancelled off.

If you let the expression to be equal to xx and then cube it, then you will get the equation 109x=x310-9x=x^3 and from here, by observation , we can conclude that the answer has to be 11.

The other two roots have to be complex conjugates, but the answer in real numbers is 11.

Vilakshan Gupta - 2 years, 12 months ago

Log in to reply

@Vilakshan Gupta Ok. Now I understood. So you have solved it Vilakshan Gupta

Ram Mohith - 2 years, 12 months ago

Can you give explanation for it? Why isn't this a real number? Principal Cube root of real number should be real.

Vaibhav Priyadarshi - 2 years, 12 months ago

Log in to reply

@Vaibhav Priyadarshi First tell is it correct or not

Ram Mohith - 2 years, 12 months ago

Log in to reply

@Ram Mohith A lot of algebra went into this, but i can't get rid of the complex part.

Michael Mendrin - 2 years, 12 months ago

Log in to reply

@Michael Mendrin Let me try another way to do this

Michael Mendrin - 2 years, 12 months ago

@Ram Mohith I think not.

Vaibhav Priyadarshi - 2 years, 12 months ago

The answer is 1.

Vilakshan Gupta - 2 years, 12 months ago

Log in to reply

Yes it is correct!

Vaibhav Priyadarshi - 2 years, 12 months ago

How is it "1"? This expression is a root of a 4th degree polynomial with no real roots.

Michael Mendrin - 2 years, 12 months ago

Log in to reply

@Michael Mendrin Let (5+213)1/3(5+2\sqrt{13})^{1/3} + (5213)1/3(5-2\sqrt{13})^{1/3} = xx

Cubing both sides, we get

10+3(52(213)2)1/3x10+3(5^2-(2√13)^2)^{1/3}x = x3x^3

109x=x310-9x=x^3

It have 1 as a solution.

Vaibhav Priyadarshi - 2 years, 12 months ago

Log in to reply

@Vaibhav Priyadarshi Why have you posted the solution when you have posted the question

Ram Mohith - 2 years, 12 months ago

Log in to reply

@Ram Mohith Sir have asked me how it is 1, so I have given the reason.

Vaibhav Priyadarshi - 2 years, 12 months ago

Log in to reply

@Vaibhav Priyadarshi But the problem now is who solved this question and who will post the next one.

Ram Mohith - 2 years, 12 months ago

Log in to reply

@Ram Mohith Vilakshan Gupta

Vaibhav Priyadarshi - 2 years, 12 months ago

@Vaibhav Priyadarshi Also I didn't understand how you got 9x

Ram Mohith - 2 years, 12 months ago

Log in to reply

@Ram Mohith By using a3+b3+3ab(a+b)a^3+b^3+3ab(a+b)

Here a+b=x.

Vaibhav Priyadarshi - 2 years, 12 months ago

Log in to reply

@Vaibhav Priyadarshi What about ab. When they both are multiplied they give a complex number

Ram Mohith - 2 years, 12 months ago

@Vaibhav Priyadarshi (52(213)2))13 (5^2-(2\sqrt{13})^2))^{\frac{1}{3}} is complex. You slipped up on sign. Or at least about fractional powers of 1-1.

Let me try to see if I can save this....

Michael Mendrin - 2 years, 12 months ago

Log in to reply

@Michael Mendrin Is (27)1/3(-27)^{1/3} not real?

Vaibhav Priyadarshi - 2 years, 12 months ago

Log in to reply

@Vaibhav Priyadarshi Unfortunately, no. This is where it gets complicated.

Michael Mendrin - 2 years, 12 months ago

Log in to reply

@Michael Mendrin Please explain me why.

Vaibhav Priyadarshi - 2 years, 12 months ago

@Michael Mendrin I agree that the principal root of (27)13(-27)^{\frac13} is an imaginary number , but we are concerned about the real valued root , and that is , 3-3.

Check this

Vilakshan Gupta - 2 years, 12 months ago

Vaibhav shall I update the note as Vilakshan Gupta solved your question. What do you think?

Ram Mohith - 2 years, 12 months ago

Consider an arbitrary polynomial X3+aX+bR[X]X^{3}+aX+b\in\mathbb{R}[X]. Then, provided Δ:=(a/3)3+(b/2)2>0\Delta:=(a/3)^{3} + (b/2)^{2} > 0, letting

C:=b/2+Δ,D:=b/2ΔC:=-b/2+\sqrt{\Delta},\quad D:=-b/2-\sqrt{\Delta}

one has (taking real cubed roots) that C1/3+D1/3C^{1/3}+D^{1/3} is the unique real root of X3+aX+bX^{3}+aX+b.

Now choosing b:=10b:=-10 and a:=9a:=9, one has 5+213=5+52=C5+2\sqrt{13}=5+\sqrt{52}=C and 5213=552=D5-2\sqrt{13}=5-\sqrt{52}=D (one may obtain the values of a,ba,b by simply solving b/2=5-b/2=5 and Δ=52\Delta=52. Thus the expression in the problem, x˘:=(5+213)1/3+(5213)1/3\breve{x}:=(5+2\sqrt{13})^{1/3}+(5-2\sqrt{13})^{1/3}, is equal to the unique real root of

X3+9X10X^{3}+9X-10

Now, note that 11 is a root of this polynomial. Thus it is the unique real root. Hence x˘=1\boxed{\breve{x}=1}.

R Mathe - 2 years, 12 months ago

Log in to reply

Good. Now you too solved this question but you cannot post the new question because Vilakshan Gupta already solved it. But this solution comes into your account.

Ram Mohith - 2 years, 12 months ago

Log in to reply

@Ram Mohith If we rewrote the expression to (5+213)13(5+213)13(5+2\sqrt{13})^{\frac{1}{3}}-(-5+2\sqrt{13})^{\frac{1}{3}}, then, yes, the value becomes 11. But what is done is to assume that (a)1n=(a)1n(-a)^{\frac{1}{n}}=-(a)^{\frac{1}{n}}, which is not true for all n>1n>1. That's not a safe step to take when doing computations, because what that does is to "convert a complex quantity into a real".

This is a controversal subject, involving fractional powers of complex numbers, which are multi-valued, and this sort of thing crops up on Brilliant from time to time.

Michael Mendrin - 2 years, 12 months ago

@Ram Mohith @Ram Mohith But this solution comes into your account. I have no idea what this sentence means. Regardless. I solved it for the sake of solving it. I don’t pay attention to who does what first nor do I look at other people’s solutions until I’ve done things myself.

@Michael Mendrin I worked completely in R\mathbb{R}. For odd nNn\in\mathbb{N} the unique real nnth root of a-a does coincide with (a1/n)-(a^{1/n}). I do appreciate that if we work in C\mathbb{C} it is all a matter of convention: there is no compelling reason to choose one argument over another (ie the choice of kZk\in\mathbb{Z} in θ+2πkn\frac{\theta+2\pi k}{n}).

R Mathe - 2 years, 12 months ago

Log in to reply

@R Mathe Well, right, what do we do if we have multiple values for this sort of thing? Just like just because we know that (2)(2)=4(-2)(-2)=4 doesn't mean it's safe to assume that 4=2\sqrt{4}=-2. Likewise fractional powers of negative numbers. Whenever I see something like that arise, it's never safe to "convert a complex into a real".

What's the value of (1)13(-1)^{\frac{1}{3}}?

I meant to explore this more in detail this morning, but this Question Game moves on.

Michael Mendrin - 2 years, 12 months ago

Log in to reply

@Michael Mendrin The value of (1)13(-1)^{\frac{1}{3}} should be (1)13(-1)^{\frac{1}{3}}, which denotes a multiple-valued complex quantity. It's kind of analogous to some quantities in quantum physics, huh? "It can have a range of values, depending on how you look at it".

Michael Mendrin - 2 years, 12 months ago

Log in to reply

@Michael Mendrin It is not a multivalued object (except maybe in set theory). In algebra, it’s just ill-defined. And in complex analysis one acknowledges the existence of many inverses, and the consequences of this being any choice of the nnth root function f:zCznexp(ıarg(z)+2πkn)f:z\in\mathbb{C}\mapsto \sqrt[n]{|z|}\exp(\imath\frac{\arg(z)+2\pi k}{n}) is holomorphic except along a semi-infinite line.

R Mathe - 2 years, 12 months ago

Log in to reply

@R Mathe "In algebra, it' s just ill-defined" (!). Well, isn't that what I've been trying to argue?

Why is my value 14(1+313+i(393))\frac{1}{4}(1+3\sqrt{13}+i(\sqrt{39}-\sqrt{3})) any less valid than "1"?

Michael Mendrin - 2 years, 12 months ago

Log in to reply

@Michael Mendrin Being ill-defined here means there be no single-valued (inverse) function f:CCf:\mathbb{C}\longrightarrow\mathbb{C}. Being multi-valued means a function of the form f:C(C)f:\mathbb{C}\longrightarrow\wp(\mathbb{C}). Both of these are technically true. But in algebra, it is not sensible to work with the latter.

Your result is fine. The whole point of this discussion, was you disputing mine. I defended my result by stating that I’m working in the framework R\mathbb{R}. You’re working in the framework C\mathbb{C}. Which framework one chooses is another issue. As soon as these are fixed, the answers are what they are—within the framework R\mathbb{R} the only valid result is 11. In your framework, which is perfectly legitimate your result is one of several perfectly valid answers.

R Mathe - 2 years, 12 months ago

Log in to reply

@R Mathe Oh, I don't know how you got the idea that I was disputing your solution. My apologies. Yes, earlier, before you jumped in with your solution, I did blurt out, "let's make sure if 1 is right!" Well, it was late at night, and so I didn't get the chance to review the matter until this morning, and saw what the problem was.

This is not the first time I've run to this sort of thing in Brilliant.org, where powers of complex numbers are involved. Several problems have already been posted to explore this topic.

An annoyance that I have is that for the sake of rigor, a function of the form y=f(x)y=f(x) is, by convention, defined to be single valued. This is the reason why y=xy=\sqrt{x} by convention only yields positive yy, i.e. only the top half of the parabola. But lots of implicit functions of the form f(x,y)=0f(x,y)=0 are multi-valued in both the xx and y)y) directions. This is the reason why whenever complex quantities are involved, especially powers of them, it raises red flags for me and I move with caution, and ask myself, "okay, what's the convention with this, and in what context is this? " This is just from habit now.

Michael Mendrin - 2 years, 12 months ago

Log in to reply

@Michael Mendrin Sir, please try to solve the latest problem 3

Ram Mohith - 2 years, 12 months ago

@Michael Mendrin Not a problem. Generally any randomly chosen implicit functions will have many locally well-defined branches. There’s no mathematical preference for any. By contrast there is sensible preference however, when one demands restricting a solution to say a particular field. Nonetheless, as I said, one always needs to fix a framework and work in it. If things happen to have more solutions in a larger framework, then so be it. For example asking, whether X2+X+1X^{2}+X+1 be irreducible without specifying an algebraic structure, is simply begging for problems, as the answers is yes or no, depending on the context (if the field is R\mathbb{R}, Q\mathbb{Q}, C\mathbb{C}, F2\mathbb{F}_{2}, … or if one works in a ring, etc.).

R Mathe - 2 years, 12 months ago

@Michael Mendrin Your example with even values is irrelevant to what I was saying. I referred only to odd nNn\in\mathbb{N}. For odd powers there is only one real root.

For nNn\in\mathbb{N} odd, the map f:xRxnRf:x\in\mathbb{R}\mapsto x^{n}\in\mathbb{R} is continuous and unbound above and below, thus the image (as a continuous image of a connected set) is all of R\mathbb{R}, moreover the map is injective. Hence there is a unique Inverse in R\mathbb{R}. Thus for all aRa\in\mathbb{R} there is exactly one value bRb\in\mathbb{R} such that bn=ab^{n}=a. This is the unique real nnth root. Eg, the unique real 33rd root of 1-1 is 1-1.

it's never safe to "convert a complex into a real"

I disagree with this. What is sensible is it is never safe to not fix a framework. And fix a framework I did. There is no compulsion to work in one particular field. Since eg QRCGL(C,3)\mathbb{Q}\subseteq\mathbb{R}\subseteq\mathbb{C}\subseteq\mathrm{GL}(\mathbb{C},3)\subseteq\ldots are field extensions, one could by your logic also argue, that it’s never safe to work in C\mathbb{C} and that one should always work in some GL(C,3)\mathrm{GL}(\mathbb{C},3).

R Mathe - 2 years, 12 months ago

Log in to reply

@R Mathe Let me try to explain what I think the problem is. Suppose instead of something as complicated as the original expression, the problem asks, "what's the value of (1)13(-1)^{\frac{1}{3}}"? There is no one unique answer, unless the problem asks for the unique real root. If one argues that "1" is a valid value of the original complicated expression, I can equally argue that my complex value is equally valid.

What I am saying, in ordinary mathematical computations, one must be careful about assuming only the unique real roots of such fractional powers of negative quantities. Otherwise, I could "show" that 1=12+32i-1 = \frac{1}{2} +\frac{\sqrt{3}}{2}i, for example (and therefore -1= 3!)

Michael Mendrin - 2 years, 12 months ago

It's creative. I want to join into your discussion.

Matin Naseri - 2 years, 12 months ago

Log in to reply

You can. In fact I am encouraging everyone to join this. Try to solve any question the first and you can post a new question.

Ram Mohith - 2 years, 12 months ago

Log in to reply

log210log8125\log_2 10 - \log_8 125

log8125=log23125\log_8 125 ={\log_{2^3} 125}

13log2125\dfrac{1}{3}{\log_{2}125}, 1253=5\sqrt[3]{125}=5

Thus log25=log8125\log_2 5 ={\log_8 125}

log210\log_2 10 - log25=\log_2 5 = log2105=\log_2{\frac{10}{5}}=log22=1\log_2 2 = 1

Matin Naseri - 2 years, 12 months ago

Log in to reply

@Matin Naseri Fine but you are the third solver to that question

Ram Mohith - 2 years, 12 months ago

Log in to reply

@Ram Mohith What I should do?

Also I have fixed my code,My last line had Errors.

Matin Naseri - 2 years, 12 months ago

Log in to reply

@Matin Naseri No problem. Leave your solution. It will come into your account. If you solved another question you will have 2 solutions and will be top in the leader board. Try to solve the latest question : Problem - 3

Ram Mohith - 2 years, 12 months ago

Problem 3:\text{Problem 3:}


In the ABC\triangle ABC ,OO is any arbitrary point in the interior of the triangle. Lines OAOA , OBOB , OCOC are drawn such that the angles OABOAB,OBCOBC and OCAOCA are each equal to ω\omega , then prove that cotA+cotB+cotC=cotω\cot A+\cot B+\cot C=\cot \omega and csc2A+csc2B+csc2C=csc2ω\csc^2A+\csc^2B+\csc^2C=\csc^2\omega

Vilakshan Gupta - 2 years, 12 months ago

Log in to reply

Ok. It is tough but no problem. But from next time onwards try to post problems of Easy and Medium difficulty.

Ram Mohith - 2 years, 12 months ago

Let a,b,ca,b,c be the angles of the triangle and xx be the angle common to all 3 vertices. Then using Law of Sines, we have

Sin(x)OB=Sin(bx)OA\dfrac{Sin(x)}{OB}=\dfrac{Sin(b-x)}{OA}
Sin(x)OC=Sin(cx)OB\dfrac{Sin(x)}{OC}=\dfrac{Sin(c-x)}{OB}
Sin(x)OA=Sin(ax)OC\dfrac{Sin(x)}{OA}=\dfrac{Sin(a-x)}{OC}

Hence

4(Sin(x))3=4Sin(ax)Sin(bx)Sin(cx)4{ \left( Sin(x) \right) }^{ 3 }=4Sin(a-x)Sin(b-x)Sin(c-x)

And from there on, using product-to-sum and sum-to-product trig identities, we have

3Sin(x)Sin(3x)=Sin(2a+x)+Sin(2b+x)Sin(2(a+b)x)Sin(3x)3Sin(x)-Sin(3x)=Sin(2a+x)+Sin(2b+x)-Sin(2(a+b)-x)-Sin(3x)

then after rearrangement and adding the term Sin(2ax)-Sin(2a-x) on both sides

Sin(2ax)+Sin(x)+Sin(2(a+b)x)Sin(2b+x)=Sin(2ax)2Sin(x)+Sin(2a+x)-Sin(2a-x)+Sin(x)+Sin(2(a+b)-x)-Sin(2b+x)=-Sin(2a-x)-2Sin(x)+Sin(2a+x)

then

4Cos(x)Sin(a)Sin(b)Sin(a+b)+4Cos(a)Sin(b)Sin(a+b)Sin(x)=-4Cos(x)Sin(a)Sin(b)Sin(a+b)+4Cos(a)Sin(b)Sin(a+b)Sin(x)= 4Cos(b)Sin(a)Sin(a+b)Sin(x)+4Cos(a+b)Sin(a)Sin(b)Sin(x)-4Cos(b)Sin(a)Sin(a+b)Sin(x)+4Cos(a+b)Sin(a)Sin(b)Sin(x)

and finally

Sin(a)Sin(b)Sin(c)Sin(x)(Cot(a)+Cot(b)+Cot(c)Cot(x))=0Sin(a)Sin(b)Sin(c)Sin(x)(Cot(a)+Cot(b)+Cot(c)-Cot(x))=0

Michael Mendrin - 2 years, 12 months ago

Log in to reply

Good solution sir. If it is correct you are the one who can post the next question. But let's see what Vilaskhan Gupta says.

Ram Mohith - 2 years, 11 months ago

Can you explain your first step, from where did the first equation come?

Vilakshan Gupta - 2 years, 11 months ago

Log in to reply

@Vilakshan Gupta I'll put that first step in, see my solution

Michael Mendrin - 2 years, 11 months ago

Log in to reply

@Michael Mendrin Vilakshan Gupta is Micheal Mendrin solution correct or not. Please tell I have to update the note.

Ram Mohith - 2 years, 11 months ago

Log in to reply

@Ram Mohith Ram, the full solution is going to be much longer than what I've posted, many more steps inbetween. What I've posted are the key steps, especiallly the middle part. I've not been able to figure out a shorter way to prove it. Maybe Gupta can just post a new question and start over.

Michael Mendrin - 2 years, 11 months ago

Log in to reply

@Michael Mendrin The solution seems fine, mine is also a similar solution. Then according to the rules of this game, you can post a new question Sir.

Vilakshan Gupta - 2 years, 11 months ago

Log in to reply

@Vilakshan Gupta Ok.Now the question is solved by Michael Mendrin sir and he can now post the latest questions.

Ram Mohith - 2 years, 11 months ago

Two unit disks are randomly thrown on the plane. If they overlap, what's the expected distance between their centers?

Michael Mendrin - 2 years, 11 months ago

Problem 4:\text{Problem 4:}


Two unit disks are randomly thrown on the plane. If they overlap, what's the expected distance between their centers?

Michael Mendrin - 2 years, 11 months ago

Log in to reply

1

Vaibhav Priyadarshi - 2 years, 11 months ago

Log in to reply

Ah, that's what's wrong with this problem, if I say your answer is wrong, then somebody else will probably figure out the correct answer.

Michael Mendrin - 2 years, 11 months ago

I solved it like this: If they overlap, maximum distance between their centres would be 2(if they just touches each other). Minimum distance between their centres would be 0, when they are coincident. So expected distance can be average of maximum and minimum which is 1.

Vaibhav Priyadarshi - 2 years, 11 months ago

Log in to reply

@Vaibhav Priyadarshi Okay, well, no, 1 is not the correct answer. In fact, if you do a computer simulation, randomly throwing unit discs on the plane, you won't get 1.

Michael Mendrin - 2 years, 11 months ago

The answer is 0\boxed{0}.

Tom Clancy - 2 years, 11 months ago

Log in to reply

@Tom Clancy The minimum and maximum possible distances are 0 and 2, so the expected value is somewhere inbetween.

Michael Mendrin - 2 years, 11 months ago

Log in to reply

@Michael Mendrin Okay, thus I think the final answer is 0.5\boxed{0.5}.

What about 0.5\boxed{0.5}?

Tom Clancy - 2 years, 11 months ago

Log in to reply

@Tom Clancy In mathematics, you don't get to guess. But, I can tell you it's some kind of a fraction.

It's not 0.5 and it's not 1/2

Pick a fraction between 0 and 2.

Michael Mendrin - 2 years, 11 months ago

I know two methods of doing it, but I know none!

One method is by using an excel sheet and other is by integration (both of which I don't know)

Just another guess....0.67

Vilakshan Gupta - 2 years, 11 months ago

Log in to reply

Yes, the correct answer is 2/3

Michael Mendrin - 2 years, 11 months ago

Log in to reply

@Michael Mendrin Sir,you may ask another question since this was solved by none...

Vilakshan Gupta - 2 years, 11 months ago

Sir, I will mark this as solved by Vilakshan Gupta. I am giving chance for Mohammad Farhan to post the latest problem.

Ram Mohith - 2 years, 10 months ago

I think this game lacks number of solvers

Ram Mohith - 2 years, 11 months ago

Log in to reply

Yes, probably.

Michael Mendrin - 2 years, 11 months ago

Log in to reply

76 members have seen this note but only 6 people are participating.

Ram Mohith - 2 years, 11 months ago

Log in to reply

@Ram Mohith Make it 7

Mohammad Farhat - 2 years, 10 months ago

Log in to reply

@Mohammad Farhat Are you interested in participating ?

Ram Mohith - 2 years, 10 months ago

Log in to reply

@Ram Mohith Yes!

Mohammad Farhat - 2 years, 10 months ago

Log in to reply

@Mohammad Farhat Ok young dynamite now you can post the problem 5. Before that do you know the regulations of this game. If not read them in the above note.

Ram Mohith - 2 years, 10 months ago

Log in to reply

@Ram Mohith Thank you.

Mohammad Farhat - 2 years, 10 months ago

Log in to reply

@Mohammad Farhat do you have any question ready

Ram Mohith - 2 years, 10 months ago

Log in to reply

@Ram Mohith Urmm.. See it https://brilliant.org/profile/mohmmad-syrhn9/sets/level-2/516970/problem/alice-needs-help/

Mohammad Farhat - 2 years, 10 months ago

Log in to reply

@Mohammad Farhat You should give any link. You should write the question in a separate new comment. Also, try to post a problem which is not from community problems.

Ram Mohith - 2 years, 10 months ago

Log in to reply

@Ram Mohith Oh OK

Mohammad Farhat - 2 years, 10 months ago

If many people start to participate then this game would be really creative.

Ram Mohith - 2 years, 11 months ago

Yes,Maybe. but How to we should solve the issue?

Matin Naseri - 2 years, 11 months ago

Log in to reply

Do you have any idea ?

Ram Mohith - 2 years, 10 months ago

Log in to reply

@Ram Mohith Let's all reshare this so that more people can see it

Mohammad Farhat - 2 years, 10 months ago

Log in to reply

@Mohammad Farhat Probably. But the problem is all those who will see may not participate in this.

Ram Mohith - 2 years, 10 months ago

Log in to reply

@Ram Mohith Let's try that first

Mohammad Farhat - 2 years, 10 months ago

Where is the 5th problem

Mohammad Farhat - 2 years, 10 months ago

Log in to reply

It is not posted till because nobody solved question 4.

Ram Mohith - 2 years, 10 months ago

Problem 5: (Solved By Ram Mohith)

There were 5 friends namely Beatricia, Jing Kai, Emmanuel, Freddy and Benjamin.

Beatricia : 9 sweets

Jing Kai: 7 sweets

Benjamin : ? sweets

Emmanuel : 8 sweets

Freddy : 6 sweets

What is ?.

(LaTeX is hard)

Mohammad Farhat - 2 years, 10 months ago

Log in to reply

7 sweets or 5 sweets. I had both explanations. I you tell the correct answer I will post the relevant one.

Ram Mohith - 2 years, 10 months ago

Log in to reply

WRONG! YOU LOSE A LIFE! Sorry TWO LIVES! 1 MORE TRY

Mohammad Farhat - 2 years, 10 months ago

Log in to reply

@Mohammad Farhat Just Kidding

Mohammad Farhat - 2 years, 10 months ago

The correct answer is 88 sweets.

Ram Mohith - 2 years, 10 months ago

Log in to reply

Explain

Mohammad Farhat - 2 years, 10 months ago

Log in to reply

@Mohammad Farhat I have posted my solution. I forgot to tell one thing. Please write the question number (here Problem 5) at the starting of your question.

Ram Mohith - 2 years, 10 months ago

Log in to reply

@Ram Mohith OK

Mohammad Farhat - 2 years, 10 months ago

Solution for Problem 5

The number of letters in each person's name is the count of sweets they have.

BeatriciaJing KaiEmmanuelFreddyBenjamin
97868\boxed{8}

Ram Mohith - 2 years, 10 months ago

Log in to reply

That's Right! That's what the Gifted Education Programme in Singapore has

Mohammad Farhat - 2 years, 10 months ago

You are now tied With Vilakshan Gupta

Mohammad Farhat - 2 years, 10 months ago

Log in to reply

But Vilakshan Gupta answer two questions which are not posted by himself. But I answered one question which I itself posted and now your question. So, first priority is given to him.

Ram Mohith - 2 years, 10 months ago

Log in to reply

@Ram Mohith OH

Mohammad Farhat - 2 years, 10 months ago

Ram, You forgot to follow (Combinatorics) and (Electricity and Magnetism)

Mohammad Farhat - 2 years, 10 months ago

Log in to reply

Because those chapters are not started yet in our academics.

Ram Mohith - 2 years, 10 months ago

Log in to reply

Oh

Mohammad Farhat - 2 years, 10 months ago

But do you know permutations?

Mohammad Farhat - 2 years, 10 months ago

Log in to reply

@Mohammad Farhat Yes some basics in our class 10.

Ram Mohith - 2 years, 10 months ago

Log in to reply

@Ram Mohith Then at least learn more fundamentals and make your fundamentals strong as you said

Mohammad Farhat - 2 years, 10 months ago

Problem - 6\text{Problem - 6}


A ball falls on surface from 10 m10~m height and rebounds to 2.5 m2.5~m. If the duration of contact with floor is 0.010.01 seconds. Find the average acceleration during the contact ?

(A)2100 m/s2(A) 2100~m/s^2

(B)1400 m/s2(B) 1400~m/s^2

(C)700 m/s2(C) 700~m/s^2

(D)400 m/s2(D) 400~m/s^2

Ram Mohith - 2 years, 10 months ago

Log in to reply

Wow, you are even kinder. You gave options

Mohammad Farhat - 2 years, 10 months ago

How many times can we pick the options?

Mohammad Farhat - 2 years, 10 months ago

Log in to reply

It not based on how many times you attempt. The first one who gives the correct solution with explanation is said to be as the first solver.

Ram Mohith - 2 years, 10 months ago

(D) because 10/2.5 =4

according to acceleration = m/s^2

so 4/0.01=400

Mohammad Farhat - 2 years, 10 months ago

Log in to reply

No. wrong explanation.

Ram Mohith - 2 years, 10 months ago

I think the correct answer is 2100ms22100{\frac{m}{s^2}} but I'm not sure about it.

Note: you have a typo in your first line. Find no fid

Tom Clancy - 2 years, 10 months ago

Log in to reply

Yes you are correct. But you should provide your explanation.

Ram Mohith - 2 years, 10 months ago

Log in to reply

@Ram Mohith Again I'm not sure about my solution.

2.5×10=25;250.01=25002.5×10=25;{\frac{25}{0.01}}=2500

10÷2.5=4;40.01=40010÷2.5= 4; {\frac{4}{0.01}}=400

2500400=21002500 - 400 = 2100

Hence the answer is 2100ms22100{\frac{m}{s^2}}

Tom Clancy - 2 years, 10 months ago

Log in to reply

@Tom Clancy Okay but why you multiplied and divided 2.5 and 10

Ram Mohith - 2 years, 10 months ago

Log in to reply

@Ram Mohith Is my solution right?

Tom Clancy - 2 years, 10 months ago

Log in to reply

@Tom Clancy Yes it is correct. Tom Clancy you have solved this question so you can now post a new question.

Ram Mohith - 2 years, 10 months ago

Log in to reply

@Ram Mohith Okay, I will post my problem as soon as possible, but is it your own problem?

Tom Clancy - 2 years, 10 months ago

Log in to reply

@Tom Clancy Tom Clancy when are you going to post your problem.

Ram Mohith - 2 years, 9 months ago

Log in to reply

@Ram Mohith I have been waiting for a new question....

Mohammad Farhat - 2 years, 1 month ago

@Ram Mohith, When will @Tom Clancy post a new question

Mohammad Farhat - 2 years, 8 months ago

Log in to reply

Don't know. As he is a new user he came to brilliant only 2 or 3 days and then he stopped.

Ram Mohith - 2 years, 8 months ago

Log in to reply

@Ram Mohith Maybe you post a new question

Mohammad Farhat - 2 years, 8 months ago

Log in to reply

@Mohammad Farhat I am now in vacation and out of station and will not be available for 2 days more. So I cannot post a problem now.

Ram Mohith - 2 years, 8 months ago

Log in to reply

@Ram Mohith ohh! Relax on your vacation

Mohammad Farhat - 2 years, 8 months ago

@Ram Mohith @Ram Mohith, can you please post another question?

Mohammad Farhat - 2 years, 7 months ago
×

Problem Loading...

Note Loading...

Set Loading...